LSAT and Law School Admissions Forum

Get expert LSAT preparation and law school admissions advice from PowerScore Test Preparation.

 Administrator
PowerScore Staff
  • PowerScore Staff
  • Posts: 8917
  • Joined: Feb 02, 2011
|
#101072
Complete Question Explanation

Strengthen. The correct answer choice is (B).

Answer choice (A):

Answer choice (B): This is the correct answer choice.

Answer choice (C):

Answer choice (D):

Answer choice (E):

This explanation is still in progress. Please post any questions below!
 netherlands
  • Posts: 136
  • Joined: Apr 17, 2013
|
#10617
Hi there PS,

I've noticed that in some MSS questions, that should actually be super easy, the answer choice is almost nothing more than a restatement of what was said in the stimulus. How do you categorize these? It's just weird bc I feel like there's a tendency to look for new additional information that strengthens the stimulus but that's not always the case. So sometimes ill actually overlook answer choices looking for something new and additional rather than seeing the answer choice that is almos a restatement of what the stimulus said, perhaps with different verbiage, and is the correct answer choice.
Last edited by netherlands on Wed Sep 04, 2013 11:47 am, edited 1 time in total.
User avatar
 Dave Killoran
PowerScore Staff
  • PowerScore Staff
  • Posts: 5852
  • Joined: Mar 25, 2011
|
#10630
Hey Netherlands,

Good to hear from you. If you have a Most Strongly Supported question stem, remember that this falls into the First Family, which is based on finding an answer choice proven by the statements in the stimulus. So, if you see a simple restatement--and it does happen--take it immediately. Sometimes, certain questions on the LSAT are just easy, and you ran across one of those. If you had a Strengthen question like this one, then you would be looking for new information, but in a Most Strongly Supported question that's not your task.

Please let me know if that helps. Thanks!
 netherlands
  • Posts: 136
  • Joined: Apr 17, 2013
|
#10683
Ok that makes sense! But just realized that the question stem actual said "which one of the following supports the hypothesis". ( My mistake :-D - I said it was an MSS!) - so that would prob make it more of a strengthening question, correct? In that case does anything change from what you just told me, or are there still cases even with this type of question stem that I'm going to see a restatement.

The superprep explanation said that it supports it ( in my mind, strengthens it) because it conforms to the stimulus. I kind of hate when they do the restatement thing, bc it completely throws me off! But I know I've seen it in other questions too.
 Steve Stein
PowerScore Staff
  • PowerScore Staff
  • Posts: 1153
  • Joined: Apr 11, 2011
|
#10887
Hey netherlands,

That one is indeed a Strengthen question, which means that new information can be presented in the correct answer choice.

Keep in mind that the question asks which one of the following, if true, supports...

So, the right answer choice will says something which, when added to the premises from the stimulus, will strengthen the author's conclusion.

I hope that's helpful! Please let me know whether this is clear--thanks!

~Steve
 Khodi7531
  • Posts: 116
  • Joined: Mar 14, 2018
|
#47761
So can anyone help with this question? Not sure if i've seen one like this before - at least the answer choice, maybe I have but in a different form.


I got rid of B and did POE to C - which I didn't realize what it was saying so circled it since I was lost - but I got rid of B because I was thinking very conditionally. I thought copying curves requires development of copying angles. I was reading it in a way like "don't reverse it".

But looking at it now, i'm assuming this goes more with normal logic and not LSAT logic - meaning, if you have learned the skills to copy angles you COULD copy curves. Which is technically an illegal reversal but that's what I mean by normal logic and not LSAT thinking.



Is that right? How do I know when this sort of thinking is required for the question. Under time I was trying to do it all mechanical and would have crossed off B no matter what. This is my only way of justifying B being correct.
User avatar
 Dave Killoran
PowerScore Staff
  • PowerScore Staff
  • Posts: 5852
  • Joined: Mar 25, 2011
|
#49172
Hi Khodi,

Thanks for the question! Hmm, I wouldn't agree with your analysis based on my reading of it, to be honest. Let's think about what the stimulus is saying:

  • "...the skills involved in copying curves must be developed before the skills involved in copying angles can be developed."
So, according to this psychologist, drawing curves is a necessary condition to drawing angles:

  • Copy Angle :arrow: Copy Curve
Now, answer choice (B) reinforces that conditional relationship by showing that indeed the kids copying angles have the ability (as expected) to copy curves. Here's an analogy:

  • A baseball writer posited that before a player can hit a home run, they must be able to swing a bat.

    Answer choice (B): All of the baseball players who can hit home runs can swing a bat.
That's ultra-simplified (and you could substitute many other things for Swing a Bat), but it gets the idea across.

Thanks!

Get the most out of your LSAT Prep Plus subscription.

Analyze and track your performance with our Testing and Analytics Package.